Derive the Principle of Least Action from the Path Integral?

In summary, the author is looking at how to derive the principle of least action from the path integral, and concludes that it can be done by starting from the ikonal approximation of the Schrödinger equation. This approximation leads in leading order to the Hamilton-Jacobi partial differential equation, which is equivalent to the action principle.
  • #1
hyksos
37
12
TL;DR Summary
Path Integral relationship to Principle of Least Action
Several weeks ago I had considered the question as to how one can start from the Schroedinger Equation, and after several transformations, derive F=ma as a limiting case. At some point in my investigations of this derivation, it occurred to me that this is simply too much work. While in principle, this derivation is entirely possible, it is very long and has to be to performed in stages. I concluded this is too much to tackle in one go.

(in rough terms, you would go from from Schroedinger Eq --> Path Integral --> Least Action --> Euler-Lagrange Eq --> Lagrangian --> ##\left(\frac{\delta \mathcal{L}}{\delta q} \ =\frac{d}{dt} mv\right)## --> ##F=ma## )

Instead of tackling this monumental task in one swoop, I have decided to make a thread here on just one stage. I want to see how to derive the classical form of the principle of Least Action by starting from the Path Integral. (Path Integral --> Least Action) At first we notice that PI does not imply POLA directly. Instead, POLA is what falls out when you take a limit of PI. Thus POLA acts as a "limiting case" of the more general PI.

I found the following form of PI, and it contains h-bar in several places. Although I disagree with the use of L(q,qdot) since that could cause problems.

$$\bra{q_{F} \ }\ket{e^{-i\hat{H} t/\hbar } |q_{I}} =\int _{\begin{array}{ c } q( t) =q_{F}\\
q( 0) =q_{I}\end{array}} Dq\ \ exp\left[ \ \ \ \frac{i}{\hbar }\int ^{t}_{0} dt^{\prime } L( q,\dot{q} \ )\right]$$

From there we want to take the limit as ##\hbar \ \rightarrow 0## . The justification is that for large objects like trees and basketballs, ##\hbar## can be approximated as if it were ##\hbar## = zero when considering their aggregate number of constituent particles. The goal is to reach this equality

$$\delta \int ^{t_{2}}_{t_{1}} L( q,\dot{q} ,t) dt\ =\ 0$$

The best response is to link to this derivation already completed by someone else , either somewhere on the PhysicsForums or offsite. Thanks.
 
Physics news on Phys.org
  • #2
The idea is that since ##exp[\frac{i}{\hbar} S]## oscillates wildly in the limit of ##\hbar \rightarrow 0##, only the set of paths on which ##S## is stationary should contribute. Thus the classical limit is that the physical trajectories ##q(t)## should satisfy ##\delta S[q(t)] = 0##. This is similar to how, under certain conditions, $$\lim_{\omega\rightarrow\infty}\int f(t) e^{i \omega t} dt = 0$$.
This reasoning is not rigorous, however I believe it can be made rigorous by analytically continuing the path integral to imaginary time so one has something along the lines of ##exp(-\frac{1}{\hbar} S)##, which has much nicer convergence properties as ##\hbar \rightarrow 0##.
 
  • Like
Likes radium and atyy
  • #3
The way via the Schrödinger equation is the socalled WKB method, i.e., the ikonal approximation of the Schrödinger equation, which leads in leading order to the Hamilton-Jacobi partial differerntial equation which is equivalent to the action principle and finally Newton's ##F=m a##.
 
  • Like
Likes msumm21 and atyy
  • #4
vanhees71 said:
The way via the Schrödinger equation is the socalled WKB method, i.e., the ikonal approximation of the Schrödinger equation, which leads in leading order to the Hamilton-Jacobi partial differerntial equation which is equivalent to the action principle and finally Newton's ##F=m a##.

Do you know where I could find this worked out in details?
 
  • #5
A very nice treatment about the WKB method (particularly about the issue how to treat the "classical turning points") is in Landau&Lifshitz vol. 3.
 
  • #6
HomogenousCow said:
The idea is that since exp[iℏS] oscillates wildly in the limit of ℏ→0, only the set of paths on which S is stationary should contribute.

Here is one treatment to tease out Schrodinger
And nobody has said the magic words "principle of stationary phase"
 

1. What is the Principle of Least Action?

The Principle of Least Action is a fundamental principle in physics that states that a physical system will follow the path of least action, which is the path that minimizes the action integral. This principle is used to describe the motion of particles and fields in classical mechanics and quantum mechanics.

2. What is the Path Integral?

The Path Integral is a mathematical tool used in quantum mechanics to calculate the probability of a particle moving from one point to another. It is based on the idea that a particle can take all possible paths between two points, and the probability of its final position is determined by the sum of all these paths.

3. How is the Principle of Least Action derived from the Path Integral?

The Principle of Least Action can be derived from the Path Integral by considering all possible paths that a particle can take between two points, and finding the path that minimizes the action integral. This involves using the Lagrangian formalism and applying the Euler-Lagrange equation to find the path of least action.

4. What are the applications of the Principle of Least Action and the Path Integral?

The Principle of Least Action and the Path Integral have various applications in physics, including classical mechanics, quantum mechanics, and field theory. They are used to describe the motion of particles and fields, calculate probabilities, and make predictions about physical systems.

5. Are there any limitations to the Principle of Least Action and the Path Integral?

While the Principle of Least Action and the Path Integral are powerful tools in physics, they do have limitations. They are most applicable to systems with well-defined paths and can be difficult to apply to systems with complex interactions or non-linear dynamics. Additionally, the Path Integral is based on the assumption that all possible paths are equally likely, which may not always be the case.

Similar threads

Replies
3
Views
1K
  • Quantum Physics
Replies
15
Views
2K
Replies
1
Views
749
Replies
1
Views
643
Replies
19
Views
2K
  • Quantum Physics
Replies
1
Views
2K
Replies
12
Views
2K
Replies
1
Views
966
Replies
6
Views
2K
Replies
33
Views
3K
Back
Top